LSAT and Law School Admissions Forum

Get expert LSAT preparation and law school admissions advice from PowerScore Test Preparation.

 lanereuden
  • Posts: 147
  • Joined: May 30, 2019
|
#66169
Lucas Moreau wrote:A is incorrect because the reasoning says that, despite quantum theory's counterintuitive consequences, it should be accepted because its predictions were shown to be inaccurate. The number of counterintuitive consequences is not at issue, nor do we know if quantum theory actually does have less counterintuitive consequences than its competitors.

C is incorrect because quantum theory's counterintuitive consequences continue to be considered counterintuitive, even after its predictions are shown to be accurate. There is nothing to indicate that quantum theory's counterintuitive consequences have been shown to be any less counterintuitive as a result of its predictions being shown to be accurate.

D is incorrect because this principle is simply not stated. It is not that quantum theory should not be rejected because it had not been subject to attempts to disprove it, it is that quantum theory should not be rejected because the attempts to disprove it have failed. The mere fact of there being serious attempts to disprove it does not affect whether or not a theory should be rejected.

E is incorrect because quantum theory's predictions have not been proven by experiment, they have merely failed to be disproven. And moreover, this fact is not the only reason quantum theory should be accepted, or at least, that is not demonstrated by the stimulus.

I hope that helps!
Okay can we revisit Why A is wrong:
First: you say A is incorrect because the reasoning says that, despite quantum theory's counterintuitive consequences, it should be accepted because its predictions were shown to be inaccurate.
In my mind it says, despite attempts to prove it inaccurate which is not necessarily the same as counterintiuve consequences, as attempts could be made to verify the validity of certain truths.

Second: , you say it should be accepted because it’s prediction were shown to be inaccurate (I think this should read accurate, not inaccurate)

Third: if we take this stimulus as sufficient assumption (as opposed to necessary assumption), then I do not think it is relevant that: “nor do we know if quantum theory actually does have less counterintuitive consequences than its competitors.” This would only matter in the case of necessary assumption question, I think.

Fourth: if we assume that fewer counterintiuve consequences merit acceptance then could that not justify the argument?
 Rachael Wilkenfeld
PowerScore Staff
  • PowerScore Staff
  • Posts: 1419
  • Joined: Dec 15, 2011
|
#67164
Hi lanereuden,

Our stimulus says that despite many attempts to show quantum theory was inaccurate, it was generally shown to be accurate within an acceptable margin of error. The results, generally showing that quantum theory works, was superior to the results of competing theories.

For answer choice (A), we know nothing about the number of counterintuitive consequences of other theories. We can't apply this principle to the stimulus because we don't know if quantum theory has fewer counterintuitive consequences.

This is neither a sufficient nor necessary assumption question, it's a question of which principle strengthens. The way that the answer choice is phrased it says IF it has fewer counterintuitive consequences than competitors then it should be accepted. But as I said, we don't know if quantum theory has fewer counterintuitive consequences. The sufficient isn't satisfied, so we don't necessarily have the necessary condition.

The correct answer choice (B) says that if a theory has withstood many serious attempts to disprove it, it should be accepted. This would strengthen because we know quantum theory has withstood many attempts, and it supports the conclusion that we should accept it.

Hope that helps!
Rachael
 Kelly R
  • Posts: 35
  • Joined: May 08, 2020
|
#83179
Hi PS,

Quick question about B. The stimulus suggests that "despite rigorous attempts to show that quantum theory's predictions were inaccurate, they were shown to be accurate within the generally accepted statistical margin of error." This does not, however, seem to suggest that quantum theory has withstood ALL serious attempts to disprove it, especially considering the fact that the results were consistent with a Margin of Error, which would allow for the theory not to withstand ALL attempts to disprove it while still maintaining overall accuracy. I accordingly eliminated B and selected what seemed like the second-best answer, E.
 Robert Carroll
PowerScore Staff
  • PowerScore Staff
  • Posts: 1819
  • Joined: Dec 06, 2013
|
#83642
Kelly,

The stimulus is claiming that quantum physics withstood all the serious attempts so far, and answer choice (B) is referring to the past tense as well, so that's no objection to the answer. The stimulus also thinks that having the deviations within the margin of error constitutes withstanding the challenge - otherwise, the theory wouldn't have survived even challenges in the past. If we take its margin of error to indicate that there's some room for a future challenge to succeed and strike down quantum physics, that's still fine - answer choice (B) is about all the serious attempts that have happened so far.

If answer choice (B) were instead "A scientific theory should be accepted if it will survive all serious attempts to disprove it," then it would be an inadequate answer, because we don't know how it will survive future challenges.

Answer choice (E) cannot possibly help the argument. It is a conditional:

a theory should be accepted :arrow: its predictions have not been disproven by experiment

Regardless of what the necessary condition said, this answer cannot ever help the argument. Because the conclusion of the argument ("accept the theory") is in the sufficient condition of this conditional, a statement like this is saying "In order for the conclusion to be true, something else is necessary." How could that help? If the "something else" is false, then, via the contrapositive, that would weaken the argument. If the "something else" is true, then we have the necessary condition of a conditional true...which gets us nowhere. To say "If the necessary is true, then the sufficient must be true" is to make the mistake of a Mistaken Reversal.

This is a point that generalizes and is especially helpful in Strengthen questions. If the answer choice is a conditional that has the conclusion's concept as the sufficient condition, that answer cannot be helpful. If the conclusion concept is necessary, or the NEGATION of the conclusion is sufficient, then the answer may help, and we have to evaluate the details to see whether it helps. Answer choice (B), for instance, is a conditional that has the conclusion concept as its necessary condition. The sufficient condition's being true in the stimulus is why answer choice (B) is helpful, and thus the correct answer.

Robert Carroll

Get the most out of your LSAT Prep Plus subscription.

Analyze and track your performance with our Testing and Analytics Package.